LSAT and Law School Admissions Forum

Get expert LSAT preparation and law school admissions advice from PowerScore Test Preparation.

 brcibake
  • Posts: 55
  • Joined: Jul 19, 2017
|
#39554
I completely ruled B out when on this question. I thought D was a strong answer. I see were B could be strong but why what makes it the strongest answer and what makes D a weaker answer?
Thank you
 nicholaspavic
PowerScore Staff
  • PowerScore Staff
  • Posts: 271
  • Joined: Jun 12, 2017
|
#39595
Hi brci,

Answer Option (D) is an Opposite Answer. "The very high prices of bulbs in the seventeenth-century Dutch tulip market were not irrational" is the exact opposite of what Mackay's thesis was, namely that there was a "speculative bubble." Here, as is the case with many Opposite Answers, the LSAC has snuck a "not" in the answer in order to confuse the test taker. Remember, always read carefully.

Answer (B) declaring that prices were " determined by speculation" is the superior choice here because it conforms quite closely to the line beginning “According to Mackay…” (lines 23–26) and is therefore the only answer choice that passes our Fact Test.

Thanks for the great question and I hope this helped!
 jaclyn.s
  • Posts: 9
  • Joined: Sep 20, 2019
|
#68295
I do not understand why answer choice D is an opposite answer. Lines 47-49 say "But this does not mean that the high prices of original bulbs are irrational."

I thought that the author was saying the original prices were determined by speculation but that they were not necessarily irrational because of the potential long-term return on investment.

Maybe I misunderstood the passage. Can you please help me to figure out where I went wrong ? Thank you! :)
 James Finch
PowerScore Staff
  • PowerScore Staff
  • Posts: 943
  • Joined: Sep 06, 2017
|
#68306
Hi Jaclyn,

You're correct that we don't know that (D) is necessarily an opposite answer, in the sense that it is directly contradicted by what Mackay is actually arguing, but it is, like (E), something that Garber is saying. The passage says that Garber is challenging Mackay's views, so as a likely inference, we can say it's probable that (D) is something that Mackay would disagree with.

Hope this helps!

Get the most out of your LSAT Prep Plus subscription.

Analyze and track your performance with our Testing and Analytics Package.